Bomba de cadena 'dispositivo de movimiento perpetuo' [duplicado]

¿Alguien podría explicarme por qué esta 'máquina de movimiento perpetuo' no funcionaría? La única explicación en la que he pensado es que sería imposible suspender el agua de una manera en la que las bolas aún pudieran viajar a través de ella...

idea

(la idea es que las bolas de un lado se levantan (en el agua) y del otro lado se dejan caer, se mueven debido a flotaciones constantemente diferentes)

Respuestas (3)

En el punto donde las bolas entran al agua desde el fondo, experimentarán una "flotabilidad negativa" que anula exactamente cualquier trabajo realizado por las bolas que flotan en el agua; para entrar en la columna de agua, debe trabajar contra la presión causada por toda la columna de agua.

Por tus respuestas, supongo que también tienes experiencia en ingeniería... :)
@Countto10 físico de formación, ingeniero de profesión.
Mi siguiente pregunta es ¿la flotabilidad de una bola que sale del líquido no cancelaría la flotabilidad negativa de una que entra al mismo tiempo?
@LucasMartin no: la presión en la parte inferior es más alta que en la parte superior, por lo que se debe hacer más trabajo para ingresar al líquido.

Puedes imaginar una puerta con forma de iris que se abre cuando la bola en la parte inferior intenta subir. Pero la máquina no funcionará porque la presión del agua empujará la pelota hacia abajo. No permitirá que la pelota entre desde abajo. Recuerde que en este punto la pelota no está completamente adentro y hay una fuerza neta hacia abajo debido al diferencial de presión entre el agua y el aire.

Escribí esta respuesta cuando floris publicó la suya. Es esencialmente lo mismo.
Creo que tu respuesta usa un vocabulario más accesible.

Como Arthur implica a continuación, mi respuesta es un argumento (erróneo) a favor del movimiento perpetuo, en lugar de en contra.

Desequilibrio de peso. Cuantas más esferas uses para lograr una mayor flotabilidad, más te quedará en el lado izquierdo, por lo que eventualmente se llegará a un equilibrio, sin importar la cantidad de esferas involucradas.

No eso no es. La flotabilidad a la derecha y el peso a la izquierda es exactamente lo que haría que esto funcionara.
@Arturo. Gracias, tiene toda la razón y necesito volver a los principios básicos, o pensar antes de responder.